300 Followers Problem - Polynomial Differential Reciprocal Summations!

Algebra Level 5

Let P ( x ) = z = 0 2015 a z x z \large{P(x) = \displaystyle \sum_{z=0}^{2015} a_z x^z} be a polynomial with real coefficients a z a_z defined by a z = ln ( e + z ) a_z = \ln(e+z) having simple roots r 1 , r 2 , , r 2015 r_1, r_2, \ldots, r_{2015} . Evaluate the value of the following upto three correct places of decimals:

k = 0 2014 ( i = 1 2015 r i k 1 P ( r i ) ) = ? \large{\sum_{k=0}^{2014} \left(\sum_{i=1}^{2015} \dfrac{r_i^{k-1}}{P'(r_i)} \right) = \ ?}


The answer is -0.868.

This section requires Javascript.
You are seeing this because something didn't load right. We suggest you, (a) try refreshing the page, (b) enabling javascript if it is disabled on your browser and, finally, (c) loading the non-javascript version of this page . We're sorry about the hassle.

1 solution

Satyajit Mohanty
Sep 13, 2015

Instead of 201 5 t h 2015^{th} degree, let us generalize the problem for n t h n^{th} degree.

Consider the function:

Q ( x ) = i r i k P ( x ) P ( r i ) ( x r i ) x k Q(x) = \sum_{i} \dfrac{r_i^k P(x)}{P'(r_i)(x-r_i)} - x^k

where P ( x ) P(x) is a n t h n^{th} degree polynomial with simple roots r 1 , r 2 , r 3 , , r n r_1, r_2, r_3, \ldots , r_n and k k is any integer with 0 k ( n 1 ) 0 \leq k \leq (n-1) . Since ( x r i ) P ( x ) (x-r_i)|P(x) for all i i , the function Q ( x ) Q(x) is a polynomial of degree (at most) n 1 n-1 . For any root r j r_j ,

Q ( r j ) = i j r i k P ( r j ) P ( r i ) ( r j r i ) + lim x r j r j k P ( x ) P ( r j ) ( x r j ) r j k Q(r_j) = \sum_{i \neq j} \dfrac{r_i^k P(r_j)}{P'(r_i)(r_j - r_i)} + \lim_{x \to r_j} \dfrac{r_j^k P(x)}{P'(r_j)(x-r_j)} - r_j^k = lim x r j ( r j k P ( r j ) ) ( P ( x ) p ( r j ) x r j ) r j k = r j k P ( r j ) P ( r j ) r j k = 0 = \lim_{x \to r_j} \left(\dfrac{r_j^k }{P'(r_j)} \right) \left( \dfrac{P(x) - p(r_j)}{x-r_j} \right) - r_j^k = \dfrac{r_j^k P'(r_j)}{P'(r_j)} - r_j^k = 0

Therefore Q ( x ) = 0 Q(x) = 0 for n n distinct values r 1 , r 2 , , r n r_1, r_2, \ldots, r_n , but since Q ( x ) Q(x) has a degree of at most n 1 n-1 , it must be identically zero. Thus:

i r i k P ( x ) P ( r i ) ( x r i ) x k = 0 \sum_{i} \dfrac{r_i^k P(x)}{P'(r_i)(x-r_i)} - x^k = 0

and, if x r i x \neq r_i for all i i , we have;

i r i k P ( r i ) ( x r i ) = x k P ( x ) . . . ( 1 ) \sum_{i} \dfrac{r_i^k}{P'(r_i)(x-r_i)} = \dfrac{x^k}{P(x)} \qquad ...(1)

For k = 0 k=0 in ( 1 ) (1) , we have:

i 1 P ( r i ) ( x r i ) = 1 P ( x ) \sum_{i} \dfrac{1}{P'(r_i)(x-r_i)} = \dfrac{1}{P(x)}

and by setting x = 0 x=0 , we have i 1 P ( r i ) r i = 1 P ( 0 ) = 1 a 0 \displaystyle \sum_{i} \dfrac{1}{P'(r_i)r_i} = -\dfrac{1}{P(0)} = -\dfrac{1}{a_0}

For k = n 1 k=n-1 in ( 1 ) (1) , we have:

i r i n 1 P ( r i ) ( x r i ) = x n 1 P ( x ) and i ( r i n 1 P ( r i ) ) ( x x r i ) = x n P ( x ) \sum_{i} \dfrac{r_i^{n-1}}{P'(r_i)(x-r_i)} = \dfrac{x^{n-1}}{P(x)} \quad \text{and} \quad \sum_{i} \left(\dfrac{r_i^{n-1}}{P'(r_i)} \right) \left(\dfrac{x}{x-r_i} \right) = \dfrac{x^n}{P(x)}

By taking the infinite limit:

lim x i ( r i n 1 P ( r i ) ) ( x x r i ) = lim x x n P ( x ) and i ( r i n 1 P ( r i ) ) = 1 a n \lim_{x \to \infty} \sum_{i} \left(\dfrac{r_i^{n-1}}{P'(r_i)} \right) \left(\dfrac{x}{x-r_i} \right) = \lim_{x \to \infty} \dfrac{x^n}{P(x)} \quad \text{and} \quad \sum_{i} \left(\dfrac{r_i^{n-1}}{P'(r_i)} \right) = \dfrac{1}{a_n}

Additionally for 0 < k < n 2 0 < k < n-2 (that is 1 < k + 1 < n 1 1 < k+1 < n-1 ) in ( 1 ) (1) , we have:

i r i k + 1 P ( r i ) ( x r i ) = x k + 1 P ( x ) \sum_{i} \dfrac{r_i^{k+1}}{P'(r_i)(x-r_i)} = \dfrac{x^{k+1}}{P(x)}

and for x = 0 x=0 , we have, i r i k + 1 P ( r i ) ( r i ) = 0 \displaystyle -\sum_{i} \dfrac{r_i^{k+1}}{P'(r_i)(r_i)} = 0 and i r i k P ( r i ) = 0 \displaystyle \sum_i \dfrac{r_i^k}{P'(r_i)} = 0 .

We conclude that:

i r i k P ( r i ) = { 1 a 0 if k = 1 0 if 0 k n 2 1 a n if k = n 1 \large{\sum_i \dfrac{r_i^k}{P'(r_i)} = \begin{cases} \color{#3D99F6}{-\dfrac{1}{a_0} \quad \text{ if } k=-1} \\ 0 \qquad \text{ if } 0 \leq k \leq n-2 \\ \color{#D61F06}{\dfrac{1}{a_n} \qquad \text{ if } k = n-1} \\ \end{cases} }

We will sum upto the asked limit, and we also know that a z = ln ( e + z ) a_z = \ln(e + z) . We have:

k = 0 2014 ( i = 1 2015 r i k 1 P ( r i ) ) = 1 a 2015 1 a 0 = 1 ln ( e + 2015 ) 1 ln ( e ) 0.868 \large{\sum_{k=0}^{2014} \left(\sum_{i=1}^{2015} \dfrac{r_i^{k-1}}{P'(r_i)} \right) = \dfrac{1}{a_{2015}} - \dfrac{1}{a_0} = \dfrac{1}{\ln(e+2015)} - \dfrac{1}{\ln(e)} \approx \boxed{-0.868}}

one of the hardest problems in brilliant, i had to resort to technology for this. would you elaborate the limits.

Aareyan Manzoor - 5 years, 7 months ago

0 pending reports

×

Problem Loading...

Note Loading...

Set Loading...